If 2^x =30 find 2^(x+3) A)8 B)5 C)240 D)200 E)250 (Good Luck! Plz solve fast!)

Answers

Answer 1

Answer:

C

Step-by-step explanation:

So we already know that:

[tex]2^x=30[/tex]

And we want to find the value of:

[tex]2^{x+3}[/tex]

So, what you want to do here is to separate the exponents. Recall the properties of exponents, where:

[tex]x^2\cdot x^3=x^{2+3}=x^5[/tex]

We can do the reverse of this. In other words:

[tex]2^{x+3}=2^x\cdot 2^3[/tex]

If we multiply it back together, we can check that this statement is true.

Thus, go back to the original equation and multiply both sides by 2^3:

[tex]2^x(2^3)=30(2^3)\\[/tex]

Combine the left and multiply out the right. 2^3 is 8:

[tex]2^{x+3}=30(8)\\2^{x+3}=240[/tex]

The answer is C.

Answer 2

Answer:

the answer is c

Step-by-step explanation:


Related Questions

Consider the following. x = t − 2 sin(t), y = 1 − 2 cos(t), 0 ≤ t ≤ 2π Set up an integral that represents the length of the curve. 2π 0 dt Use your calculator to find the length correct to four decimal places.

Answers

Answer:

L = 13.3649

Step-by-step explanation:

We are given;

x = t − 2 sin(t)

dx/dt = 1 - 2 cos(t)

Also, y = 1 − 2 cos(t)

dy/dt = 2 sin(t)

0 ≤ t ≤ 2π

The arc length formula is;

L = (α,β)∫√[(dx/dt)² + (dy/dt)²]dt

Where α and β are the boundary points. Thus, applying this to our question, we have;

L = (0,2π)∫√((1 - 2 cos(t))² + (2 sin(t))²)dt

L = (0,2π)∫√(1 - 4cos(t) + 4cos²(t) + 4sin²(t))dt

L = (0,2π)∫√(1 - 4cos(t) + 4(cos²(t) + sin²(t)))dt

From trigonometry, we know that;

cos²t + sin²t = 1.

Thus;

L = (0,2π)∫√(1 - 4cos(t) + 4)dt

L = (0,2π)∫√(5 - 4cos(t))dt

Using online integral calculator, we have;

L = 13.3649

The Airline Passenger Association studied the relationship between the number of passengers on a particular flight and the cost of the flight. It seems logical that more passengers on the flight will result in more weight and more luggage, which in turn will result in higher fuel costs. For a sample of 21 flights, the correlation between the number of passengers and total fuel cost was 0.668.


(1)
State the decision rule for 0.10 significance level: H0: Ï â‰¤ 0; H1: Ï > 0 (Round your answer to 3 decimal places.)


Reject H0 if t >
(2)
Compute the value of the test statistic. (Round your answer to 3 decimal places.)


Value of the test statistic

Answers

Answer:

Decision Rule:  To reject the null hypothesis if t > 1.328

t = 3.913

Step-by-step explanation:

The summary of the given statistics include:

sample size n = 21

the correlation between the number of passengers and total fuel cost r = 0.668

(1) We are tasked to state the decision rule for 0.10 significance level

The degree of freedom df = n - 1

degree of freedom df = 21 - 1

degree of freedom df = 19

The  null and the alternative hypothesis can be computed as:

[tex]H_o : \rho < 0\\ \\ Ha : \rho > 0[/tex]

The critical value for [tex]t_{\alpha, df}[/tex]  is  [tex]t_{010, 19}[/tex] = 1.328

Decision Rule:  To reject the null hypothesis if t > 1.328

The test statistics can be computed as follows by using the formula for t-test for Pearson Correlation:

[tex]t = r*\sqrt{ \dfrac{(n-2)}{(1-r^2)}[/tex]

[tex]t = 0.668*\sqrt{ \dfrac{(21-2)}{(1-0.668^2)}[/tex]

[tex]t = 0.668*\sqrt{ \dfrac{(19)}{(1-0.446224)}[/tex]

[tex]t = 0.668*\sqrt{ \dfrac{(19)}{(0.553776)}[/tex]

[tex]t = 0.668*5.858[/tex]

t = 3.913144

t = 3.913    to 3 decimal places

An octagonal pyramid ... how many faces does it have, how many vertices and how many edges? A triangular prism ... how many faces does it have, how many vertices and how many edges? a triangular pyramid ... how many faces does it have, how many vertices and how many edges?

Answers

1: 8 faces and 9 with the base 9 vertices and 16 edges

2: 3 faces and 5 with the bases 6 vertices and 9 edges

3: 3 faces and 4 with the base 4 vertices and 6 edges

Hope this can help you.

what number has 7 ten thousands, 1 thousand, 1 hundred, and no ones?

Answers

Answer:

[tex]71,100[/tex]

Step-by-step explanation:

If you are trying to find a number that is written in word form, we can just use place values to find what goes where.

A number is broken down into this:

Ten thousands, thousands, hundreds, tens, ones.

If they have 7 ten thousands, the first digit will be a 7.

If they have 1 thousand, the second digit will be a 1.

If they have 1 hundred, the third digit will be a 1.

Since nothing is stated about tens, we assume it's value is 0.

And since there are no ones, it's value is 0.

So:

71,100.

Hope this helped!

These girts stasts jogging from the same point around
acircular track and they complete one round in 24
Seconds 36 seconds and 48 seconds respectively,
After.
how much time will they meet atone point?​

Answers

Answer:

2hrs 24mins

Step-by-step explanation:

Very simple the time they will meet again at the point will be the LCM of their various time taken to complete a cycle.

Ans LCM(24, 36, 48) = 144 mins

= 2hrs 24mins

Answer:

The answer is 2 hours and 24 minutes

Step-by-step explanation:

Hope you get this right:)

How to find which ratio is largest

Answers

In order to check whether its greater than or less than, First You have to make denominator value equal of both ratio value. then we have to make denominator value equal i.e. After that you check which ratio Numerator value is greater.

Which of the following is the correct set notation for the set of perfect squares between 1 and 100 (including 1 and 100)?
Select the correct answer below:

{p2∣p∈ℤ and 1≤p≤10}

{p2∣p∈ℤ and 1


{p2∣p∈ℝ and 1≤p≤10}

{p2∣p∈ℤ and 1

Answers

Answer:

[tex]\{P^2: P\ E\ Z\ and\ 1\leq p\leq 10\}[/tex]

Step-by-step explanation:

Given

Range: = 1 to 100 (Inclusive)

Required

Determine the notation that represents the perfect square in the given range

Represent the range with P

P = 1 to 100

Such that the perfect squares will be and integers

In set notation, integers are represented with Z

The set notation becomes

[tex]\{P^2: P\ E\ Z\ and\ 1\leq p\leq 10\}[/tex]

The [tex]\leq[/tex] shows that 1 and 100 are inclusive of the set

Consider the following functions. f={(−1,1),(1,−2),(−5,−1),(5,3)} and g={(0,2),(−3,−4),(1,−2)} Step 1 of 4: Find (f+g)(1).

Answers

Answer:

  -4

Step-by-step explanation:

(f+g)(1) = f(1) +g(1)

In each case, you need to locate the ordered pair with 1 as the first element.

  (1, f(1)) = (1, -2) . . . . f(1) = -2

  (1, g(1)) = (1, -2) . . . . g(1) = -2

  f(1) +g(1) = (-2) +(-2) = -4

(f+g)(1) = -4

Henry is investing at a continuously compounded annual interest rate of 4.5%. How many years will it take for the balance
to triple? Round your answer up to the nearest whole number, and do not include the units in your answer.

Answers

Answer:

1 year

Step-by-step explanation:

Hello,

Continuously compounding with an annual interest rate of 4.5% means multiplying the initial investment by (for t tears).

[tex]\displaystyle e^{(1+4.5\%)t}=e^{\left( 1.045\cdot t \right) }[/tex]

So we need to find t so that:

[tex]\displaystyle e^{\left( 1.045\cdot t \right) }=3\\\\1.0.45t=ln(3)\\\\t=\dfrac{ln(3)}{1.045}=1.051304...[/tex]

Rounding to the nearest whole number gives 1 year.

Hope this helps.

Do not hesitate if you need further explanation.

Thank you

Answer:

25

Step-by-step explanation:

Trust me

Suppose that 80% of all registered California voters favor banning the release of information from exit polls in presidential elections until after the polls in California close. A random sample of 25 registered California voters is selected.

Required:
a. Calculate the mean and standard deviation of the number of voters who favor the ban.
b. What is the probability that exactly 20 voters favor the ban?

Answers

Answer:

a. Mean = 20

Sd = 4

b. Probability of X = 20 = 0.1960

Step-by-step explanation:

we have

n = 25

p = 80% = 0.8

mean = np

= 0.8 * 25

= 20

standard deviation = √np(1-p)

= √25*0.8(1-0.8)

=√4

= 2

probability that exactly 20 favours ban

it follows a binomial distribution

= 25C20 × 0.8²⁰ × 0.2⁵

= 53130 × 0.01153 × 0.00032

= 0.1960

Probability of X = 20 = 0.1960

Aaron wants to mulch his garden. His garden is x^2+18x+81 ft^2 One bag of mulch covers x^2-81 ft^2 . Divide the expressions and simplify to find how many bags of mulch Aaron needs to mulch his garden.

Answers

Answer:

Step-by-step explanation:

Given

Garden: [tex]x^2+18x+81[/tex]

One Bag: [tex]x^2 - 81[/tex]

Requires

Determine the number of bags to cover the whole garden

This is calculated as thus;

[tex]Bags = \frac{x^2+18x+81}{x^2 - 81}[/tex]

Expand the numerator

[tex]Bags = \frac{x^2+9x+9x+81}{x^2 - 81}[/tex]

[tex]Bags = \frac{x(x+9)+9(x+9)}{x^2 - 81}[/tex]

[tex]Bags = \frac{(x+9)(x+9)}{x^2 - 81}[/tex]

Express 81 as 9²

[tex]Bags = \frac{(x+9)(x+9)}{x^2 - 9\²}[/tex]

Evaluate as difference of two squares

[tex]Bags = \frac{(x+9)(x+9)}{(x - 9)(x+9)}[/tex]

[tex]Bags = \frac{(x+9)}{(x - 9)}[/tex]

Hence, the number of bags is [tex]Bags = \frac{(x+9)}{(x - 9)}[/tex]

Based on the dot plots shown in the images, which of the following is a true statement? A. Set B has the greater mode. B. Set A has more items than set B. C. Set A is more symmetric than set B. D. Set B has the greater range.

Answers

D. Set B has the greater range.

the temp fell 3 degrees every hour for 5 hours what's the change in temperature​

Answers

Answer:

-15

Step-by-step explanation:

If it fell 3 deg every hour for 5 hours so the equation is 3*5 plus a - sign because it dropped degrees

Using the power series methods solve the 1st order Lane-Emden Equation:
xy = 2y + xy = 0
You may only use a power series solution to find both linearly independent functions. This means you may not use Abel’s theorem, variation of parameters or reduction of order.

Answers

Answer:

Step-by-step explanation:

xy = 2y + xy = 0

Hence, 2y + xy = 0 ---------(1)

Differentiating equation (1) n times by Leibnitz theorem, gives:

2y(n) + xy(n) + ny(n - 1) = 0

Let x = 0: 2y(n) + ny(n - 1) = 0

2y(n) = -ny(n - 1)

∴ y(n) = -ny(n - 1)/2 for n ≥ 1

For n = 1: y = 0

For n = 2: y(1) = -y

For n = 3: -3y(2)/2

For n = 4: -2y(3)

A clothing business finds there is a linear relationship between the number of shirts, n, it can sell and the price, p, it can charge per shirt. In particular, historical data shows that 1,000 shirts can be sold at a price of $30, while 3,000 shirts can be sold at a price of $10. Find a linear equation in the form p(n)=mn+b that gives the price p they can charge for n shirts.

Answers

Answer:

p(n) = -1/100 n  40

Step-by-step explanation:

Use the two points (n, p): (1000, 30) and (3000, 10).

Now we find the equation of the line that passes through these two points.

m = (10 - 30)/(3000 - 1000)

m = -20/2000

m = -1/100

p(n) = mn + b

30 = -1/100 * 1000 + b

30 = -10 + b

b = 40

The equation is:

p(n) = -1/100 n  40

A technician is testing light bulbs to determine the number of defective bulbs. The technician records the table below to show the results. Result of Light Bulb Test Number of Bulbs Tested 14 28 84 336 Number of Defective Bulbs Found 1 2 6 ? The technician expects to find 24 defective bulbs when 336 are tested. Which statement explains whether the technician’s reasoning is correct, based on the information in the table?

Answers

Answer:

He should find 24 defective lightbulbs.

Step-by-step explanation:

1. Divide the number of defective bulbs by the total number of bulbs for each section.

2. Make sure the number you get is the same each time.

3. Divide the guessed number of bulbs (24) by the total number of bulbs (336)

4. If the number you got for step 4 matches the number you got for step 3, then he is right

Answer:

The answer is A

Step-by-step explanation:

on NCCA

How many different sets of polar coordinates can be given for a point, within one rotation? I thought it was infinite, but the given options are 1, 2, 3, and 4.

Answers

Answer:

the answer is 4

Step-by-step explanation:

so 1 rotation is like a circle 1 unit circle requires 4 quadrant to be in this is the most simplified i can get

Answer:

Solution : 4

Step-by-step explanation:

The question asks us how many polar coordinates are possible for one rotation. For one rotation there will be 4 polar coordinates, one present in each quadrant such that,

( r, theta ), ( r, theta ), ( - r, theta ), ( - r, theta )

Respectively if theta was q say,

( r, q ), ( r, - q ), ( - r, q ), ( - r, -q )

Therefore there are 4 sets of polar coordinates for one rotation, in each of the 4 quadrants.

PLS HELP ASAP Solve the inequality and enter your solution as an inequality in the box below 8>4-x>6

Answers

Answer:

−4<x<−2

Step-by-step explanation:

8 > 4 − x > 6

8 > −x + 4 > 6

8 + −4 > −x + 4 + −4 > 6 + −4

4 > −x > 2

Since x is negative we need to divide everything by -1 which gives us...

−4 < x < −2

How would you simplify and rationalize this expression? [tex]\frac{5\sqrt[4]{2}}{4\sqrt[4]{162} }[/tex]

Answers

Answer:

5/12

Step-by-step explanation:

(5 * 2^1/4)/4 * 162^1/4) = (5 * 2^1/4)/4 * 3 *2^1/4)

multiply top and bottom by 2^3/4

(5 * 2)/4 * 3 * 2) = 10/24 = 5/12

A potato chip company makes potato chips in two flavors, Regular and Salt & Vinegar. Riley is a production manager for the company who is trying to ensure that each bag contains about the same number of chips, regardless of flavor. He collects two random samples of 10 bags of chips of each flavor and counts the number of chips in each bag. Assume that the population variances of the number of chips per bag for both flavors are equal and that the number of chips per bag for both flavors are normally distributed. Let the Regular chips be the first sample, and let the Salt & Vinegar chips be the second sample. Riley conducts a two-mean hypothesis test at the 0.05 level of significance, to test if there is evidence that both flavors have the same number of chips in each bag. (a) H0:μ1=μ2; Ha:μ1≠μ2, which is a two-tailed test. (b) t≈1.44 , p-value is approximately 0.167 (c) Which of the following are appropriate conclusions for this hypothesis test?
A. There is insufficient evidence at the 0.05 level of significance to conclude that Regular and Salt & Vinegar chips have different amounts of chips per bag.B. There is sufficient evidence at the 0.05 level of significance to conclude that Regular and Salt & Vinegar chips have different amounts of chips per bag.C. Reject H0.D. Fail to reject H0.

Answers

Answer:

A. There is insufficient evidence at the 0.05 level of significance to conclude that Regular and Salt & Vinegar chips have different amounts of chips per bag.

D. Fail to reject H0.

Step-by-step explanation:

From the summary of the given test statistics.

The null and the alternative hypothesis are:

[tex]H_0:\mu_1=\mu_2 \\ \\ Ha:\mu_1 \neq \mu_2[/tex]

This test is also a two tailed test.

Similarly, the t value for the test statistics = 1.44

The p- value - 0.167

The level of significance ∝ = 0.05

The objective we are meant to achieve here is to determine which of the following from the given options are appropriate conclusions for this hypothesis test.

From what we have above:

Decision Rule: We fail to reject the null hypothesis since the p-value is greater than the level of significance at 0.05

CONCLUSION: Therefore, we can conclude that  there is insufficient evidence at the 0.05 level of significance to conclude that Regular and Salt & Vinegar chips have different amounts of chips per bag as we fail to reject H0.

What is the area of a parallelogram if the coordinates of its vertices are (0, -2), (3,2), (8,2), and (5, -2)?

Answers

Answer: 20 sq. units .

Step-by-step explanation:

Let A(0, -2), B(3,2), C(8,2), and D(5, -2) are the points for the parallelogram.

First we plot these points on coordinate plane, we get parallelogram ABCD.

By comparing the y-coordinate of B and C with A and D , we get

height = 2+2 = 4 units

Also by comparing the x coordinates of A and D, we get base = 5-0= 5 units  

Area of parallelogram = Base x height

= 5 x 4 = 20 sq. units

Hence, the area of a parallelogram ABCD is 20 sq. units .

Fiona wrote the linear equation y = y equals StartFraction 2 over 5 EndFraction x minus 5.x – 5. When Henry wrote his equation, they discovered that his equation had all the same solutions as Fiona’s. Which equation could be Henry’s? x – x minus StartFraction 5 over 4 EndFraction y equals StartFraction 25 over 4 EndFraction.y = x – x minus StartFraction 5 over 2 EndFraction y equals StartFraction 25 over 4 EndFraction.y = x – x minus StartFraction 5 over 4 EndFraction y equals StartFraction 25 over 2 EndFraction.y = x – x minus StartFraction 5 over 2 EndFraction y equals StartFraction 25 over 2 EndFraction.y =

Answers

Answer:

D. [tex]x-\frac{5}{2}y = \frac{25}{2}[/tex]

Step-by-step explanation:

Given

[tex]y = \frac{2}{5}x - 5[/tex]

Required

Determine its equivalent

From the list of given options, the correct answer is

[tex]x - \frac{5}{2}y = \frac{25}{2}[/tex]

This is shown as follows;

[tex]y = \frac{2}{5}x - 5[/tex]

Multiply both sides by [tex]\frac{5}{2}[/tex]

[tex]\frac{5}{2} * y = \frac{5}{2} * (\frac{2}{5}x - 5)[/tex]

Open Bracket

[tex]\frac{5}{2} * y = \frac{5}{2} * \frac{2}{5}x - \frac{5}{2} *5[/tex]

[tex]\frac{5}{2}y = x - \frac{25}{2}[/tex]

Subtract x from both sides

[tex]\frac{5}{2}y - x = x -x - \frac{25}{2}[/tex]

[tex]\frac{5}{2}y - x = - \frac{25}{2}[/tex]

Multiply both sides by -1

[tex]-1 * \frac{5}{2}y - x * -1 = - \frac{25}{2} * -1[/tex]

[tex]-\frac{5}{2}y + x = \frac{25}{2}[/tex]

Reorder

[tex]x-\frac{5}{2}y = \frac{25}{2}[/tex]

Hence, the correct option is D

[tex]x-\frac{5}{2}y = \frac{25}{2}[/tex]

Answer:

The 4th option

Step-by-step explanation:

A soda bottling company’s manufacturing process is calibrated so that 99% of bottles are filled to within specifications, while 1% is not within specification. Every hour, 12 random bottles are taken from the assembly line and tested. If 2 or more bottles in the sample are not within specification, the assembly line is shut down for recalibration. What is the probability that the assembly line will be shut down, given that it is actually calibrated correctly? Use Excel to find the probability. Round your answer to three decimal places.

Answers

Answer:

The probability that the assembly line will be shut down is 0.00617.

Step-by-step explanation:

We are given that a soda bottling company’s manufacturing process is calibrated so that 99% of bottles are filled to within specifications, while 1% is not within specification.

Every hour, 12 random bottles are taken from the assembly line and tested. If 2 or more bottles in the sample are not within specification, the assembly line is shut down for recalibration.

Let X = Number of bottles in the sample that are not within specification.

The above situation can be represented through binomial distribution;

[tex]P(X=r)=\binom{n}{r} \times p^{r}\times (1-p)^{n-r};x=0,1,2,3,.....[/tex]

where, n = number of trials (samples) taken = 12 bottles

             x = number of success  = 2 or more bottles

            p = probabilitiy of success which in our question is probability that  

                 bottles are not within specification, i.e. p = 0.01

So, X ~ Binom (n = 12, p = 0.01)

Now, the probability that the assembly line will be shut down is given by = P(X [tex]\geq[/tex] 2)

  P(X [tex]\geq[/tex] 2) = 1 - P(X = 0) - P(X = 1)

                 = [tex]1-\binom{12}{0} \times 0.01^{0}\times (1-0.01)^{12-0}-\binom{12}{1} \times 0.01^{1}\times (1-0.01)^{12-1}[/tex]

                 = [tex]1-(1 \times 1\times 0.99^{12})-(12 \times 0.01^{1}\times 0.99^{11})[/tex]

                 = 0.00617


Factor.
x2 + 11x

x2 + 11x
x(x + 11)
11(x + 11)
0(x2 + 11x)

Answers

Answer:

x(x + 11)

Step-by-step explanation:

x^2 + 11x when factored gives a result of x(x + 11)

Answer:

x(x+11)

Step-by-step explanation:

We are given the expression:

[tex]x^2+11x[/tex]

This can be factored using the Greatest Common Factor (GCF).

The GCF of x^2 and 11x is x.

Factor out an x.

[tex]x(x+11)[/tex]

x^2+11x factored is: x(x+11).

I don't understand word problems can someone please answer it for me and I need it ASAP.

Answers

Answer:

Inequality: 3 + 1.2c

What you'd put on graph: 1 ≥ 13.50

There are 2229 students in a school district. Among a sample of 452 students from this school district, 163 have mathematics scores below grade level. Based on this sample, estimate the number of students in this school district with mathematics scores below grade level.

a. 804
b. 844
c. 884
d. 0.36

Answers

Answer:

A. 804

Step-by-step explanation:

Given the total number of students in the school to be 2229 students. If among a sample of 452 students from this school district, 163 have mathematics scores below grade level, then we can determine the number of students in this school district with mathematics scores below grade level based on the sample scores using ratio.

Let the number of students in this school district with mathematics scores below grade level be x. The ratio of the students with math score below grade level to total population will be x:2229

Also, the ratio of the sample students with math score below grade level to sample population will be 163:452

On equating both ratios, we will have;

x:2229 =  163:452

[tex]\dfrac{x}{2229} = \dfrac{163}{452}\\ \\cross\ multiplying;\\\\\\452*x = 2229*163\\\\x = \dfrac{2229*163}{452}\\ \\x = \frac{363,327}{452}\\ \\x = 803.8\\\\x \approx 804[/tex]

Hence the estimate of the number of students in this school district with mathematics scores below grade level based on the sample is 804

Which statement best describes a sequence? a.All sequences have a common difference. b.A sequence is always infinite. c.A sequence is an ordered list. d.A sequence is always arithmetic or geometric.

Answers

Answer:

C

Step-by-step explanation:

A sequence is defined as a list of numbers or objects in a special order.

They may be arithmetic or geometric or neither.

For example

0, 1, 4, 9, 16, 25, ..... ← is the sequence of square numbers.

Note it is neither arithmetic or geometric.

A school newspaper reporter decides to randomly survey 15 students to see if they will attend Tet festivities this year. Based on past years, she knows that 24% of students attend Tet festivities. We are interested in the number of students who will attend the festivities.


Find the probability that at most 4 students will attend. (Round your answer to four decimal places.)

Answers

Answer:

0.70319018

Step-by-step explanation:

Given the following:

Number of students surveyed (n) = 15

Probability of attending tet festival (p) = 24% =0.24

Therefore,

Probability of not attending (1 - p) = (1 - 0.24) = 0.76.

The probability that at most 4 students will attend can be obtained using the binomial probability relation:

p(x) = nCx * p^x * (1 - p)^(n-x)

At most 4 students means:

p(x=0) + p(x=1) + p(x=2) + p(x=3) + p(x=4)

p(x=0) = 15C0 * 0.24^0 * 0.76^(15 - 0)

p(x=0) = 1 * 1 * 0.0004701 = 0.00047018

p(x=1) = 15C1 * 0.24^1 * 0.76^(14)

p(x=1) = 15 * 0.24 * 0.021448 = 0.07721

p(x=2) = 15C2 * 0.24^2 * 0.76^(13) =

p(x=2) = 105 * 0.0576 * 0.02822 = 0.17068

p(x=3) = 15C3 * 0.24^3 * 0.76^(12)

p(x=3) = 455 * 0.013824 * 0.037133 = 0.23356

p(x=4) = 15C4 * 0.24^4 * 0.76^(11) =

p(x=4) = 1365 * 0.0033177 * 0.048859 = 0.22127

0.00047018 + 0.07721 + 0.17068 + 0.23356 + 0.22127 = 0.70319018

What is the slope of the line that passes through (2, 12) and (4, 20)?On the graph of the equation 3x + 2y = 18, what is the value of the y-intercept?

Answers

Answer: The slope of the line that passes through (2, 12) and (4, 20) is 4.

The value of the y-intercept is 9.

Step-by-step explanation:

Slope of line passing through (a,b) and (c,d) = [tex]\dfrac{d-b}{c-a}[/tex]

Then, the slope of the line that passes through (2, 12) and (4, 20) = [tex]\dfrac{20-12}{4-2}[/tex]

[tex]=\dfrac{8}{2}=4[/tex]

So, the slope of the line that passes through (2, 12) and (4, 20) is 4.

To find the y-intercept of 3x + 2y = 18, first write in slope intercept form y=mx+c ( where c= y-intercept ).

[tex]2y=-3x+18\\\\\Rightarrow\ y=-\dfrac{3}{2}x+9[/tex]

By comparison,  c= 9

Hence, the value of the y-intercept is 9.

The graph of g(x) = x – 8 is a transformation of the graph of f(x) = x. Which of
the following describes the transformation?
(A) translation 8 units down
(B) translation 8 units up
(C) translation 8 units right
(D) translation 8 units left

Answers

Technically the answer is both A and C, because you can see it as (x-8)^1, meaning 8 to the right, or just x-8, meaning 8 down.
Other Questions
An annuity provides for 30 annual payments. The first payment of 100 is made immediately and the remaining payments increase by 8 percent per annum. Interest is calculated at 13.4 percent per annum. Calculate the present value of this annuity. Why does Captain Miller get frustrated with her role in meeting with Iraqi civilians who have had family killed in the war? A. She knows that the women would rather be speaking with male soldiers. B. She wants to be in more direct battles with the enemies. C. She thinks using female soldiers only for this purpose is sexist. D. She only wants to provide medical care to U.S. military personnel Firms that compete in the global marketplace typically face two types of competitive pressures, namely, the pressures for _______ and _______. a. global integration; local responsiveness b. politically sensitivity; market leadership c. cost reductions; marginal costs d. price reductions; cost reductions To the nearest tenth, what is the value of P(C|Y)? 0.4 0.5 0.7 0.8 Which conclusion can be drawn from the experimental observation that a single strand of DNA contains 2100 dA residues and 1800 dT residues? The data set {3, 7, 5, 4, 1} consists of the lengths, in minutes, of a sample of speeches at an awards banquet. Use a formula to find the standard deviation of the sample, and label it with the correct variable. g Use the References to access important values if needed for this question. A researcher took 2.592 g of a certain compound containing only carbon and hydrogen and burned it completely in pure oxygen. All the carbon was changed to 7.851 g of CO2, and all the hydrogen was changed to 4.018 g of H2O . What is the empirical formula of the original compound need help will give 5 stars. please can someone help me solve this.. please help!! A practice, favored by unions, which contractually binds employers to hire only workers who are already members of the union is called a(n): Which observation led Mendel to the idea that traits were controlled by more than a single factor? Crossing two purple-flowered pea plants always resulted in a purple-flowered pea plant. Crossing two white-flowered pea plants always resulted in a white-flowered pea plant. A trait that was not seen in the first generation may later reappear in the second generation. A trait that was not seen in the first generation was completely gone and never reappeared. PLEASE HELP, WILL GIVE BRAINLIEST IF CORRECT!!!! (08.06 MC) Mike and his friends bought cheese wafers for $2 per packet and chocolate wafers for $1 per packet at a carnival. They spent a total of $25 to buy a total of 20 packets of wafers of the two varieties. Part A: Write a system of equations that can be solved to find the number of packets of cheese wafers and the number of packets of chocolate wafers that Mike and his friends bought at the carnival. Define the variables used in the equations. (5 points) Part B: How many packets of chocolate wafers and cheese wafers did they buy? Explain how you got the answer and why you selected a particular method to get the answer. (5 points) Dump out half of the particles. Place your hand tightly over the top and shake the container. Then remove most of the remaining particles, and shake the container again. Compared with the full container, which states of matter do these two models most closely represent? Explain. Evaluate the following expression. 8 (10) 7 1/1 What makes something that you read exciting to you and makes you want to continue reading it? Write a short story about what it might be like if you woke upone morning with amermaid tail. Which quadratic equation would be used to solve for the unknown dimensions?0 = 2w2512 = w2512 = 2w2512 = 2l + 2w If sin = 5 over 6, what are the values of cos and tan ? An 80-year-old client that takes a diuretic for the management of hypertension, informs the nurse that shetaes laxatives daily to promote bowel movements. The nurse assess the client for possible symptoms ofwhich condition? Which statement contains the best example of understatement?A. Pink cherry trees crowded the lane that ran down the center of thebig campus.B. I put on my newly laundered shirt and began to strut around like apro golfer.C. Given that she had saved my life, I decided that some thanks werein orderO D. The smell of freshly baked cinnamon rolls from across the streetwas life alteringSUBMIT